Form 25 Flashcards

1
Q
  1. A 9-year-old boy has had alopecia and hypocalcemia since birth. His serum 1;25-dihydroxycholecalciferol concentration is within the reference range: serum 24;25-dihydroxycholecalciferol is undetectable. Serum parathyroid hormone concentration is above
    the upper limit of the laboratory assay and needs to be reassayed at a dilution. The results are pending. Which of the following is the most likely cause of his condition?
    A) Loss-of-function mutation in the calcium-sensing receptor
    B) Multiple endocrine neoplasia type I
    C) Mutations inactivating the vitamin D 24-hydroxylase gene
    D ) Mutations inactivating the vitamin D receptor
A

Correct Answer: D.
Vitamin D plays a role in serum calcium and phosphate homeostasis by promoting the intestinal absorption of calcium and phosphate. Parathyroid hormone (PTH) also has a role in calcium and phosphate regulation by stimulating osteoclastic bone reabsorption
and distal convoluted tubular calcium reabsorption and phosphate excretion in the kidney Vitamin D deficiency can be caused by malabsorption in the intestinal tract, malnutrition or insufficient dietary intake, and decreased sun exposure. Decreased concentration
of vitamin D results in decreased intestinal cacium absorption and hypocalcemia, sensed by the parathyroid gland via calcium-sensing receptors, which leads to an increase in secretion of PTH to normalize serum calcium concentration. Increased PTH causes
bone resorption, liberating calcium and phosphorus, otherwise stored in bone. In this patient, 1,25-dihydroxycholecalciferol, the active form of vitamin D, is within reference range. However the vitamin D receptor must also exist and function appropriately for the
hormone to exert its effect. Vitamin ID binds a nuclear receptor, exerting its effect by gene transcription. In this case, a normal concentration of Vitamin D with persistent hypocalcemia suggests an inactivating mutation in the receptor itself, or a mutation in a
regulatory sequence that prevents transcription of the genes coding for the receptor. In turn, the patient experiences persistent hypocalcemia despite normal vitamin D concentration, as the receptor mutation prevents the action of vitamin D in promoting the
intestinal absorption of ca cium.
Incorrect Answers: A. B, and C.
Loss-of-function mutation in the calcium-sensing receptor (Choice A) in the parathyroid gland would prevent the parathyroid gland from sensing low concentrations of calcium and reacting by secreting PTH. This would cause hypoparathyroidism, not
hyperparathyroidism as seen in the patient.
Multiple endocrine neoplasia type I (Choice B) is characterized by often functional pituitary, pancreatic, and parathyroid tumors. Pituitary tumors may secrete prolactin or growth hormone, pancreatic tumors may secrete insulin, glucagon, or vasoactive intestinal
peptide, and parathyroid adenomas may secrete PTH. A functionally active parathyroid adenoma would cause an increase in PTH. and because of the role of PTH in calcium reabsorption and bone resorption, hypercalcemia would result.
Mutations inactivating the vitamin D 24-hydroxylase gene (Choice C) would result in decreased concentration of 24,25-dihydroxycholecalciferol, an inactive metabolite. Vitamin D would persist in its active form as a result, which may indirectly result in
hypercalcemia.
**Educational Objective: **Vitamin D plays a role in serum calcium and phosphate homeostasis by promoting the intestinal absorption of calcium and phosphate. Vitamin D deficiency typically leads to decreased intestinal calcium absorption, hypocalcemia, increased
PTH secretion, and increased PTH-mediated bone resorption.

How well did you know this?
1
Not at all
2
3
4
5
Perfectly
2
Q

Assuming the positive predictive value of a test is 50% and the negative predictive value is 75%; which of the following tables are consistent with these values?

A

Correct Answer: A.
Educational Objective: Positive predictive value defines the proportion of positive test results that are true positive and is calculated as TP / (TP + FP) Negative predictive value defines the proportion of negative test results that are true negative and is calculated as TN / (TN + FN).

How well did you know this?
1
Not at all
2
3
4
5
Perfectly
3
Q

A 43-year-old woman comes to the physician because of vague discomfort, nausea, easy bruising; and weight loss. Her prothrombin time is increased. The most likely cause of the increased prothrombin time is damage to which of the following cells?
A ) Endothelial cells
B) Fibroblasts
C) Hepatocytes
D ) Macrophages
E) Skeletal muscle cells
F ) T lymphocytes

A

Correct Answer: C.
C and S), complement, apoproteins, triglycerides, cholesterol, bile acids, and albumin, and play critical roles in biochemical metabolism (eg, gluconeogenesis). Many manifestations of liver disease can be attributed to dysfunction of hepatocytes, which are the
primary parenchymal cells within the liver. Prolonged prothrombin time (PT) is a common finding as a result of deficiency of factor VII, and in this case, the patient’s easy bruising reflects this acquired coagulopathy. Hypoglycemia from impaired gluconeogenesis. nausea from increased gut transit time, jaundice from hyperbilirubinemia, and hypogonadism from altered metabolism of estrogens are all common findings in patients with chronic liver dysfunction. Malnutrition and weight loss also frequently occur.
Incorrect Answers: A. B D, E, and F.
Endothelial cell damage (Choice A) results iin hypercoagulability and associated vascular disease. Endothelial cells line the inner lumen of blood vessels and function to reduce clotting. Damage to endothelial cells, which occurs via multiple mechanisms (eg.
hypertension, smoking, hyperlipidemia, hyperglycemia), results in local nitric oxide deficiency, fibrosis, inflammation, and calcification, and can lead to myocard al infarction, stroke, and peripheral vascular disease.
Fibroblast damage (Choice B) would result in disordered synthesis of the extracellular matrix. As fibroblasts are ubiquitous and exist in nearly every organ system, local deficiencies would produce variable phenotypic effects. Damage to fibroblasts iin the skin, for
example, impairs the synthesis and release of collagen thereby altering the normal process of wound healing.
Macrophage damage (Choice D) and dysfunction predisposes to infection. Disorders in the activation and/or chemotaxis of macrophages ead to an increased susceptibility to mycobacterial infections as a result of failed intracellular killing and granuloma formation.
Skeletal muscle cell damage (Choice E) is a common finding in acquired and inherited conditions such as muscular dystrophy, mitochondrial myopathy, toxic, metabolic, and inflammatory myopathies, and rhabdomyolysis. Symptoms are determined by the
mechanism of injury but result in either pain, weakness, or both affected muscles.
T lymphocyte damage (Choice F), dysfunction, or absence is typical of immunodeficiency syndromes and can be genetic or acquired. HIV/AIDS affects CD4+ T lymphocytes specifically resulting in susceptibility to a wide array of opportunistic pathogens (eg, fungal,
mycobacterial, viral, and bacterial). Iatrogenic damage to T lymphocytes such as that which occurs with cytotoxic chemotherapy also predisposes to systemic infection.
Educational Objective: Hepatocytes are critical for the synthesis of multiple coagulation factors. Dysfunction of hepatocytes results in an increased PT/INR secondary to the impaired synthesis of factor VII and may result in a predisposition to bleeding.

How well did you know this?
1
Not at all
2
3
4
5
Perfectly
4
Q

A 63-year-old man has had this gradually enlarging lesion on his right forearm for the past 3 years. Which of the following is the most likely diagnosis?
A ) Actinic keratosis
B} Blue nevus
C) Compound nevus
D ) Dermatofibroma
E) Halo nevus
F ) Hemangioma
G) Malignant melanoma
H ) Pyogenic granuloma
I ) Seborrheic keratosis
J) Squiamous cell carcinoma

A

Correct Answer: G.
Malignant melanoma is likely to be present when a lesion demonstrates asymmetry irregular appearing borders: variable coloration, a diameter greater than 6 mm. and rapid evolution in characteristics. Malignant melanoma can rapidly invade and metastasize, which carries a poor prognosis when diagnosed late. Subtypes include superficial spreading, nodular, lentigo maligna, and acral lentiginous. Any lesion with features suggestive of malignant melanoma should be surgically excised with negative margins and pathologically examined for the depth of dermal invasion.

Educational Objective: Melanoma should be suspected when lesions demonstrate asymmetry, border irregularity, variable coloration, diameter greater than 6mm, or changing features.

How well did you know this?
1
Not at all
2
3
4
5
Perfectly
5
Q

A 60-year-old man has a surgical excision of a brain mass; a photomicrograph of excised tissue is shown He has smoked 1 pack of cigarettes daily for the past 50 years. His blood pressure is 130/90 mm Hg.
Laboratory studies show polycythemia, hypercalcemia, and microscopic hematuria. Immunostaining of the tissue is positive for epithelial membrane antigen and negative for carcinoembryonic antigen. Which of the
following is the most likely site of the primary neoplasm?
A) Colon
B) Kidney
C) Liver
D) Lung
E) Lymph node
F) Prostate
G) Testis

A

Correct Answer: B.
Renal cell carcinoma (RCC) is an adenocarcinoma of tubular epithelial cells. RCC is the most common primary malignancy of the kidney and most commonly occurs in older male smokers. It can present with gross or microscopic hematuria, flank pain: weight
oss: or fever. Laboratory analysis may show polycythemia or hypercalcemia as a result of associated paraneoplastic syndrome production of erythropoietin or parathyroid hormone-related peptide. Hypercalcemia can also result from bony metastasis. Diagnosis of
RCC typically occurs with contrast-enhanced CT scan or MRI and is confirmed by biopsy at the time of nephrectomy. Biopsy of RCC is uniquely characterized by polygonal clear cells: because of the accumulation of lipid and carbohydrate content in the cells. It spreads hematogenously and commonly presents as a metastatic neoplasm. The brain and lung are frequent sites of metastasis.

Educational Objective: Renal cell carcinoma often presents in older men with a history of cigarette smoking, and the brain is a potential site of metastasis. Laboratory analysis may show hematuria along with hypercalcemia or polycythemia secondary to paraneoplastic syndromes. Histology typically demonstrates polygonal clear cells related to the accumulation of lipid and carbohydrate content in the cells.

How well did you know this?
1
Not at all
2
3
4
5
Perfectly
6
Q
  1. The biological effects of thyroid hormone and retinoic acid are mediated through similar mechanisms. Which of the following properties is common to the receptors for these two compounds?
    A) Ability to activate phospholipase C
    B) DNA-binding domain
    C) Formation of a dimer on binding of hormone
    D) Interaction with a G protein
    E) Tyrosine kinase activity
A

Correct Answer: B.
Educational Objective: Nuclear hormone receptors contain DNA-binding domains and exert their effects by serving as DNA transcription factors. Common examp es of nuclear receptors include the thyroid hormone receptor and the retinoic acid receptor

How well did you know this?
1
Not at all
2
3
4
5
Perfectly
7
Q
  1. A 57-year-old man conies to the physician because of an intermittent cough and a 9-kg (20-lb) weight loss over the past 6 months. He has smoked 1 pack of cigarettes daily for 42 years. His serum calcium concentration is 13.3 mg/dl_. An x-ray of the chest
    shows a 4-cm central mass in the left lung. Which of the following is the most likely diagnosis?
    A) Adenocarcinoma of the lung
    B) Large cell lymphoma
    C) Metastatic osteosarcoma
    D ) Metastatic renal cell carcinoma
    E) Small cell carcinoma of the lung
    F ) Squamous cell carcinoma of the lung
A

Correct Answer: F.
Squamous cell carcinoma of the lung is the second most common type of primary lung cancer after adenocarcinoma. Risk factors for all major types of lung cancer include tobacco use, secondhand smoke. asbestos: or radon exposure, and a family history of lung cancer. Features associated with squamous cell carcinoma of the lung include pulmonary cavitations: central location, and hypercalcemia because of paraneoplastic parathyroid hormone-related peptide (PTHrP) production. Histologic characteristics include
polygonal cells with intercellular bridges, eosinophilic cytoplasm, keratin pearls, and extensive necrosis. Luing cancer in general typically presents with cough, unintentional weight loss, hemoptysis, chest pain, dyspnea, and hoarseness; occasionally, wheezing, focal rhonchi or hypedrophic osteoarthropathy may be noted on examination. Diagnosis is made by chest imaging and examination of a biopsy specimen. Prognosis is a function of the cancer type along with grading and staging of the disease. It is often detected once metastatic, at which point the prognosis is poor.

Educational Objective: Centrally located primary lung cancers include squamous cell carcinoma of the lung and small cell carcinoma of the lung. Squamous cell carcinoma is the more common subtype and is associated with hypercalcemia because of
paraneoplastic PTIHrP production.

How well did you know this?
1
Not at all
2
3
4
5
Perfectly
8
Q

A 34-year-old woman is brought to the emergency department by her husband because of ain inability to see objects in the peripheral visual fields in both eyes for 1 day. Her temperature is 37.2aC (99°F), pulse is G8/min. respirations are 16/min, and blood
pressure is 124,78 mm Hg. Automated computerized perimetry examination shows a loss of the temporal visual field in both eyes. Neurologic examination shows no other abnormalities. This patient is most likely to have which of the following additional
findings?
A ) Amenorrhea
B) Aphasia
C) Double vision
D) Loss of memory
E) Seizures

A

Correct Answer: A.
This patient’s bitemporal hemianopsia is most likely the result of optic chiasm compression from a pituitary adenoma: which is often associated with amenorrhea in women secondary to the overproduction of prolactin. The pituitary gland is in the sella turcica. A
pituitary adenoma is a benign brain tumor that can present with mass effect symptoms such as headache and bitemporal hemianops a (loss of bilateral temporal visual fields). Bitemporal hemianopsia occurs because of compression of the optic chiasmi which
contains fibers from the bilateral nasal retina (containing visual information from the temporal eye fields) . Pituitary adenomas can cause symptoms from hyperpituitarism or hypopituitarism, depending on whether the tumor is functionally secreting hormones.. The
most common functional pituitary adenoma is a prolactinoma: other types of pituitary adenomas secrete follicle-stimulating hormone (FSH); luteinizing hormone (LH). adrenocorticotropic hormone (ACTH), thyroid-stimulating hormone (TSH), or growth hormone
(GH). Prolactinomas suppress gonadotropin-releasing hormone (GnRH). which results in decreased FSH and LH production and manifests as amenorrhea. Additionally if a pituitary adenoma is nonfunctional but sufficiently large: it can interfere with normal hormone secretion by the pituitary gland: causing hypopituitarism. In this setting, decreased secretion of FSH and LH could also manifest as amenorrhea.

**Educational Objective: **Pituitary adenomas can cause mass effect symptoms such as headache and bitemporal hemianopsia from compression of the adjacent optic chiasm Additionally, they may be functional or nonfunctional in nature. Hypersecretion of hormones: most commonly prolactin, can manifest with amenorrhea and galactorrhea in females.

How well did you know this?
1
Not at all
2
3
4
5
Perfectly
9
Q
  1. A 22-year-old woman is admitted to the hospital because of a 10-day history of polydipsia and polyuria. She says that the urge to urinate often awakens her at night. She has been taking lithium carbonate for 2 years for bipolar disorder: her dosage was
    increased 6 months ago because of recurrent severe manic episodes. Her vital signs are within normal limits. Physical examination shows no abnormalities. Over the next 24 hours: urine excretion totals 6.5 L. Laboratory studies at this time show a serum
    sodium concentration of 148 mEq/IL. serum osmolality of 315 mOsmol/kg, and urine osmolality of 75 mOsmol/kg. After administration of desmopressin, urine output and osmolality do not change. Which of the following mechanisms is the most likely cause of the polyuria in this patient?
    A) Decreased secretion of ADH (vasopressin)
    B) Inhibition of cAMP-mediated processes iin the collecting duct
    C) Osmotic diuresis due to increased amounts of glucose in the tubule
    D ) Polydipsia-induced washout of the renal interstitial concentration gradient
    E) Renal medullary necrosis due to decreased papillary blood flow
A

Correct Answer: B.
Nephrogenic diabetes insipidus (NDI) is commonly acquired secondary to medications or electrolyte disturbances. The pathophysiology of NDI occurs such that the renal collecting duct becomes insensitive to antidiuretic hormone (ADH) produced by the posterior pituitary. Normally. ADH triggers the insertion of aquaporin channels into the collecting duct membrane via a cAMP-mediated pathway. By consequence, when there is inhibition of this process in the collecting duct, as in NDI, the ability of the kidney to recaim free water is compromised. Dehydration results, as is demonstrated by this patient, along with symptoms of polydipsia (compensatory response to low serum volume or increased serum osmolarity) and polyuria or incontinence as a result of the large volume of dilute urine produced). NDI is an adverse effect of chronic lithium use. Lithium concentrates within the cells of the renal collecting duct and interferes with the signaling pathway related to ADH, leading to limited aquaporin insertion and resultant NDI. In severe cases of dehydration, vital signs may disclose tachycardia and hypotension. Physical examination discloses no abnormalities, apart from potential signs of volume depletion (eg, weak pulse, skin tenting). Laboratory studies show increased serum osmolarity, hypernatremia, and inappropriately dilute urine. Normally, urine should be concentrated in states of hyperosmolar serum, as the main physiologic function of ADH is to reclaim water, thereby maintaining serum osmolarity and sodium balance in addition to plasma volume. Desmopressin, an ADH analog, is used as a means of differentiating nephrogenic from central Dl. Central Dl occurs because of an absence of ADH production, not a failure of the renal collecting tubule to respond to it. If urine output and serum osmolarity decrease following desmopressin administration, a central cause of Dl (eg, head trauma, hypothalamic damage, pituitary tumor) should be investigated. No change in response to desmopressin indicates that the cause of Dl is nephrogenic. The treatment of NDI includes volume repletion plus thiazide diuretics.
**Educational Objective: **Nephrogenic diabetes insipidus is an adverse effect of lithium carbonate. Lithium impairs the ability of the collecting tubule to reclaim free water by interfering with the pathway of antidiuretic hormone in the cells. Patients typically present with polydipsia, polyuria, hyperosmolar serum, dilute urine, and failure to respond to desmopressin.

How well did you know this?
1
Not at all
2
3
4
5
Perfectly
10
Q
  1. A 77-year-old woman dies in the hospital after a long illness. Her vertebral column, obtained at autopsy is shown in the photograph. The process shown is most likely associated with an increase in which of the
    following?
    A) Calcium
    B) Estrogen
    C) lnterleukin-1 (IL-1)
    D) Monoclonal immunoglobulin
    E) Vitamin D
A

Correct Answer: C.
Osteoporosis is a common condition that is characterized by the progressive loss of bone mineral density leading to decreased bone strength. This decreased bone mineral density often leads to fragility fractures, which can greatly decrease mobility and increase the risk of death in elderly individuals. The disease classically affects postmenopausal women with inflammatory disorders, and individuals with metabolic or endocrine disorders such as hypercortisolism or hyperparathyroidism. Osteoporosis can also be induced
by long-term treatment with medications that cause an increase in bone resorption such as corticosteroids. The diagnosis is made using a DEXA scan via calculation of a T-Score. A T-score of -2.5 or less, that is a bone density measurement that is less than 2.5 standard deviations below the mean, is diagnostic of osteoporosis. T-scores between -1.0 and -2.5 are defined as osteopenia, lnterleukin-1 (IL-1) is also known as osteoclast activating factor. IL-1 leads to an increase in RANK ligand signaling and subsequent osteoclast-mediated bone resorption. Osteoporosis is thereby characterized by an increase in osteoclast number and activity, which is driven by IL-1.

**Educational Objective: **The balance of osteoblast and osteoclast interactions is key in the maintenance of bone mineral density, lnterleukin-1, also called osteoclast activating factor, can result in bone mineral density loss when increased.

How well did you know this?
1
Not at all
2
3
4
5
Perfectly
11
Q
  1. A IQ-year-old boy is brought to the physician because of a 3-day history of difficulty walking because his right foot drops when he lifts it. He is a member of a wrestling team at his high school. Physical examination shows weakness of the right ankle dorsiflexor muscles. The right ankle evertor muscles have full power. Sensation to pinprick is decreased between the great and second toes of the right foot. Sensation over the rest of the foot is normal. Which of the following nerves is most likely damaged in this patient?
    A ) Common fibular (peroneal)
    B) Deep fibular (peroneal)
    C) Sciatic
    D ) Superficial fibular (peroneal)
    E) Tibial
A

Correct Answer: B.

Educational Objective: The peripheral nerves of the leg are often sites of compression injury or entrapment causing sensory loss and motor weakness. The superficial fibular nerve everts the foot, while the deep fibular nerve dorsiflexes the foot, and the tibial nerve plantarflexes the foot. Weakness in these movements can indicate which nerve is injured.

How well did you know this?
1
Not at all
2
3
4
5
Perfectly
12
Q
  1. During a study on exercise: a 45-year-old woman ruins on a treadmill for 30 minutes. An increase in which of the following is most likely to change gastrocnemius muscle blood flow in this woman?
    A) Interstitial adenosine concentration
    B) Interstitial amino acid concentration
    C) Interstitial gIucose concentration
    D ) Interstitial oxygen tension
    E) Parasympathetic stimulation
    F ) Sympathetic stimulation
A

Correct Answer: A.
During aerobic exercise, the cellular consumption of adenosine triphosphate (ATP) increases. Myosin motor proteins dephosphorylate ATP and subsequently use the energy released to generate muscle contraction. Adenosine will become diphosphate, monophosphate, or completely without phosphate as a result of this reaction, and it must then be reconverted to ATP via the Krebs cycle and oxidative phosphorylation in the mitochondrion. Byproducts of energy metabolism regulate regional blood flow, such as carbon dioxide and organic acids. Additional y: as the interstit al adenosine concentration increases, blood vessels dilate in response. This is known as exercise hyperem a. and it occurs through mediation from local regulators. Increased acidity, adenosine: hypoxia: nitric oxide: and prostaglandins are all important mediators of exercise-induced vasodilation. Adenosine is also a vasodilator in coronary vascular smooth muscle as well as in skeletal muscle. Adenosine modulates vasodilation by binding to the adenosine receptor and increasing cyclic adenosine monophosphate (cAMP) This leads to inhibition of myosin light chain kinase and vascular smooth muscle relaxation.

**Educational Objective: **There are multiple mediators of local muscle blood flow. These include adenosine, lactate, hydrogen ions, and the part al pressures of oxygen and carbon dioxide. These autoregulatory mediators serve to increase skeletal muscle blood flow during exercise.

How well did you know this?
1
Not at all
2
3
4
5
Perfectly
13
Q
  1. A 67-year-old man is brought to the physician because of a 2-day history of double vision and drooping of his left eyelid. He has no history of trauma to the eye. His vital signs are within normal limits. Physical examination shows ptosis of the left eyelid. When the eyelid is raised manually, the eye is fixed in the out position and the pupil is dilated. The visual acuity of the left eye is within normal limits. Which of the following is the most likely cause of these findings?
    A) Aneurysm of the posterior communicating artery
    B) Compression of the superior cervical ganglion
    C) Damage to the trochlear nerve
    D ) Occlusion of the scleral venous sinus
    E) Tumor of the optic nerve
A

Correct Answer: A.
This patient’s presenting findings of unilateral ptosis; exotropia and hypotropia (inferior and lateral pupil position), and mydriasis (pupillary dilation) are consistent with an oculomotor nerve palsy. Two of the most common causes of acquired oculomotor nerve palsy are microvascular ischemia and nerve compression from an adjacent vascular aneurysm, commonly located at the posterior communicating artery (PCA). Upon leaving the brainstem, the oculomotor nerve courses beneath the posterior cerebral and posterior communicating arteries and is vulnerable to aneurysmal compression at this location. The fibers responsible for control of the pupil are found on the superficial aspect of the oculomotor nerve and are thus first affected by external compression. This distinguishes acquired oculomotor nerve palsy caused by aneurysmal external compression from palsy caused by microvascular ischemia, which typically spares the pupil. The detection of a new oculomotor nerve palsy is therefore a medical emergency, as aneurysmal rupture can lead to a life-threatening subarachnoid hemorrhage. A PCA aneurysm is diagnosed via CT angiography, MR angiography, or conventional angiography.

Educational Objective: Aneurysms of the posterior communicating artery may compress the ipsilateral oculomotor nerve, resulting in diplopia, mydriasis, exotropia, and hypotropia. They are diagnosed on CT angiography, MR angiography, or conventional angiography.

How well did you know this?
1
Not at all
2
3
4
5
Perfectly
14
Q

A 62-year-old man conies to the physician for a follow-up examination. Two weeks ago; he was discharged from the hospital after sustaining an acute myocardial infarction. He smokes 2 packs of cigarettes and drinks four 12-oz beers daily. His diet mostly consists of cured meats and fast food. He does not exercise. The patient tells the physician. “I know that I need to make some changes in how II live so that my heart can be healthier. I just don’t have the willpower to quit smoking and drinking and all that stuff right now.” Which of the following best describes this patient’s stage of behavioral change?
A ) Precontemplation
B) Contemplation
C) Preparation
D ) Action
E) Maintenance

A

Correct Answer: B.

In this scenario, the patient is aware of the potential consequences of his unhealthy habits and is considering reversing the habits. Although he has identified that his behavior is detrimental, he is indecisive about whether he is ready to act, which is consistent with the contemplation stage. The stages of behavioral change are used to define a patient’s readiness to change a health-related behavior such as substance use, diet, or exercise habits. In sequential order the stages of behavioral change are precontemplation, contemplation, preparation, action, maintenance,, and termination. Physicians aim to move patients through these stages overtime with an interview technique called motivational interviewing. Motivational interviewing involves using open-ended, non-judgmental questions to he p the patient explore their reasons for wanting to change or maintain the habit.

Educational Objective: The contemplation stage of behavioral change refers to ambivalence about changing a behavior, iin which the patient expresses knowledge of the consequences of their detrimental behavior but is not yet prepared to make plans to change. Motivational interviewing is a technique used by physicians to promote a patient’s readiness for change.

How well did you know this?
1
Not at all
2
3
4
5
Perfectly
15
Q
  1. A 35-year-old man conies to the physician because of a 2-year history of slowly progressive muscle weakness of hiis thighs. His father and paternal uncle have a disorder involving proximal muscle weakness of the lower extremities. Physical examination shows no other abnormalities. A muscle biopsy specimen shows ragged red fibers. Electron-microscopic examination of this specimen is most likely to show which of the following inclusions?
    A) Intranuclear filamentous
    B) Lysosomal
    C) Mitochondrial crystalline
    D) Paranuclear filamentous
    E) Sarcoplasmic reticular
A

Correct Answer: C.
Mitochondrial crystalline inclusions on electron microscopy iin association with proximal muscle weakness, ragged red fibers on muscle biopsy: and a compatible family history are consistent with mitochondrial myopathy. Mitochondria are the cellular organelles that produce ATP via oxidative phosphorylation, a process that requires four enzyme complexes to produce a transmembrane potential that is ultimately used to produce ATP via ATP synthase. Mitochondrial DIMA is separate from somatic nuclear DNA; and mutations in genes encoding any of these complexes can result in the failure of oxidative phosphorylation, which results in a subsequent failure to generate ATP: and consequential myopathy and neurologic symptoms. However, there are additional mutations that affect mitochondrial RMA translation, trafficking and incorporation of respiratory protein complexes, and maintenance of the inner mitochondrial membrane that can also lead to mitochondrial myopathy. As both the skeletal muscle and neurologic system require vast quantities of ATP for normal function, these are the two organ systems most common y affected by mitochondrial DNA mutations. Musculoskeletal manifestations include muscle pain: fatigue, exercise intolerance, and increased concentrations of creatine kinase, while potential neurologic symptoms may include ophthalmoplegia, seizures, myoclonus, and peripheral neuropathy. Mitochondr al diseases are strictly inherited through the mother, as sperm do not contain mitochondria to contribute to the fertilized embryo. As a result of the failure of energy metabolism, the cells may compensate by producing additional mitochondria. On muscle biopsy, these mitochondrial aggregates, when visualized on the background of the myofibers, appear as ragged red fibers. Electron microscopy demonstrates abnormal mitochondrial crystalline inclusions

Educational Objective: Mitochondrial crystalline inclusions on electron microscopy and ragged red fibers on muscle biopsy in a patient with inherited proximal muscle weakness is characteristic of mitochondrial myopathy.

How well did you know this?
1
Not at all
2
3
4
5
Perfectly
16
Q

A 49-year-old woman comes to the emergency department because of a 3-day history of fever shortness of breath, and confusion. She is a postal worker. Her temperature is 38.4X (101 2°F): respirations are 28/min, and blood pressure is 100/60 mm Hg.
Physical examination shows nuchal rigidity. Breath sounds are decreased on the right side of the clhest. A lumbar puncture is done. Analysis of cerebrospinal fluid (CSF) shows:
Glucose 18 mg/dL
138 mg/dL
638/mm3
Total protein
Leukocyte count
Segmented neutrophils
Monocytes
87%
13%
RBC 2300/mm3
A Gram stain of CSF shows large, gram-positive; spore-forming bacilli . A chest x-ray shows marked widening of the mediastinum. Which of the following extracellular virulence factors most likely enables the causal organism to evade phagocytosis?

A) Alginate
B) Glucuronoxylomannan
C) Hyaluronic acid
D ) Polyglutamic acid
E) Pollyribitol phosphate

A

Correct Answer: D.

Anthrax is the disease caused by the bacterium Bacillus anthracis, which manifests with pulmonary. cutaneous: or gastrointestinal syndromes. The hallmarks of pulmonary anthrax include cough; shortness of breath, and chest pain within weeks to months after inhalation. Pulmonary anthrax is often fatal, which may occur via hemorrhagic mediastinitis. Anthrax also causes cutaneous lesions, often painless, necrotic eschars that are generally easily treated with antibiotics. Ingestion of anthrax-infected meat can cause diarrhea and abdominal pain. The pathogen is a gram-positive rod and facultative anaerobe, which arranges in chains on microscopy. The bacterium has two principal virulence factors, which are its polyglutamic acid capsule and toxin production. The polyglutamic acid capsule serves to prevent phagocytosis. Specific toxins produced by the bacterium include lethal factor (LF) and edema factor (EF). LF leads to apoptosis of macrophages, while EF alters cyclic adenosine monophosphate signaling pathways within the immune cells.

Educational Objective: Anthrax is a pathogenic bacterium, which uses a polyglutamic acid capsule to evade phagocytosis. Many other bacteria produce polysaccharide or glycosaminoglycan capsules that act in a similar fashion to protect the bacteria and increase virulence.

How well did you know this?
1
Not at all
2
3
4
5
Perfectly
17
Q

An investigator is designing a study to compare a new behavioral program for attention-deficit/hyperactivity disorder (ADHD) with the standard behavioral modification program for this disorder. Because ADHD is more common among boys: girls and boys are
randomized into the two treatment groups separately. Which of the following types of treatment allocation is most likely being used in this study?
A ) Alternation
B) Open
C) Outcome-adapted
D ) Simple random assignment
E } Stratification

A

Correct Answer: E.
Stratification describes the allocation of subjects or the analysis of results within a trial by a factor other than the treatment given. If involves subdividing a study population into homogeneous subgroups then analyzing each subgroup in comparison to the study population or other subgroups. Advantages of this technique are that it permits analysis of subpopulations that may vary from a population at large. In this case: the researchers speculate that since ADHD is more common among boys: there may be a meaningful difference when comparing the new and standard treatment programs applied to boys as applied to girls; whereas that meaningful difference may not be detected if the combined population is analyzed. Stratification is one method of reducing confounding variables. In this case, the study population has been stratified based on gender between the new and standard interventions.

Educational Objective: Stratification describes allocating subjects or results by a factor that is not necessarily the treatment administered; groups may be stratified or subdivided based on age, gender, profession, ethnicity, or other demographic features. It is one
method to control for confounding variables.

How well did you know this?
1
Not at all
2
3
4
5
Perfectly
18
Q

A 55-year-old woman is scheduled to undergo tnansvaginaI hysterectomy and oophorectomy for dysfunctional uterine bleeding. During the procedure: the uterus must be separated from all surrounding pelvic structures. Identification and incision of which of
the following structures that attaches to the cervical region and extends posteriorly is most appropriate in this patient?
A) Mesometrium
B) Mesosalpinx
C) Mesovaiiium
D ) Ovarian ligament
E) Round ligament of the uterus
F) Uterosacral ligament

A

orrect Answer: F.
The uterosacral ligament is one of the structures that supports the uterus and holds it in place in the pelvis. It is a paired structure that connects the uterus to the sacrum at the level of the cervix. They may also be called the rectouterine ligaments or sacrocervical
igaments. The uterus is also supported by the broad ligament which attaches to the superior aspect of the uterus and is a layer of peritoneum. The pubocervical ligaments and cardinal ligaments join the uterosacral ligament in supporting the middle portion of the uterus. The inferior portion of the uterus receives further support from the muscular components of the pelvic floor such as the levator ani. Each of these connections must be severed in order to successfully remove the uterus during a hysterectomy.

Educational Objective: The uterosacral ligament is a paired structure that extends bilatera ly from the cervical region to the sacrum, helping to support the uterus in the pelvis.

How well did you know this?
1
Not at all
2
3
4
5
Perfectly
19
Q

A 1-month-old male newborn is brought to the emergency department by his mother because of a 5-day history of vomiting after feedings. His mother says the vomiting is forceful, and appears to contain only formula. She says he seems eager to feed after each episode. He appears irritable. He is at the 25th percentile for length and weight. His temperature is 37 6DC (99.7T), pulse is 140/min: respirations are 24/min: and blood pressure is 30/40 mm Hg. Physical examination shows decreased skin turgor. Which of the following sets of serum findings (in mEq/L) is most likely in this newborn at this time?

A

Correct Answer: A.
Pyloric stenosis is caused by hypertrophy of the pyloric sphincter leading to gastric outlet obstruction and vomiting. It commonly presents with nonbloody; nonbilious emesis after feedings beginning around 3 to 6 weeks of age. It is more common in male infants. Symptoms may begin with occasional vomiting, then progress to projectile emesis after every feed: dehydration, and malnutrition as the stenosis worsens. Repeated bouts of vomiting can cause metabolic and electrolyte derangements, classically hypochloremic, hypokalemic metabolic alkalosis caused by losses of hydrochloric acid, a major component of stomach secretions. Metabolic alkalosis occurs secondary to dehydration and subsequent volume contraction plus relative excess of bicarbonate. Volume depletion also
eadstothe activation of the renin-angiotensin-aldosterone system, which in turn stimulates renal tubule bicarbonate reabsorption and new bicarbonate generation. Metabolic alkalosis, volume depletion, and increased concentration of aldosterone also leads to renal potassium excretion and hypokalemia. Hypokalemia also contributes to metabolic alkalosis via transcellular exchange of extracellular sodium and protons for potassium ions. Diagnosis of pyloric stenosis is typically by abdominal ultrasonography, which demonstrates thickened pyloric musculature, and treatment requires surgical pyloromyotomy.

**Educational Objective: **Pyloric stenosis is caused by hypertrophy of the pyloric sphincter, resulting in gastric outlet obstruction, which typically first presents at around 3 to 6 weeks of life. It is characterized by repeated vomiting after feeds, leading to dehydration and a hypochloremic, hypokalemic metabolic alkalosis.

How well did you know this?
1
Not at all
2
3
4
5
Perfectly
20
Q

A 33-year-old woman with HIV infection is brought to the emergency department 30 minutes after she had a generalized tonic-clonic seizure. She also has a 2-month history of daily headaches. Physical examination shows no signs of meningismus. Muscle
strength is 3/5 in the left upper extremity and 5/5 in the right upper extremity. Her CD4+ T-lymphocyte count is 22/mm3 (N>5G0), and plasma HIV viral load is 50?000 copies/mL. A CT scan of the head shows a 3-cm lesion in the right cerebral cortex . Serologic studies show a positive IgG antibody titer to Toxoplasma gondii. Treatment with pyrimethamine and sulfadiazine is initiated. During the next 2 weeks; she has three additional seizures. Two weeks after starting antibiotic therapy a CT scan of the head shows that the lesion has increased to 3.5 cm.. Which of the following is the most likely cause of this mass?
A) Bacterial abscess
B) Cerebral toxoplasmosis
C) Glioblastoma
D) Metastatic disease
E) Non-Hodgkin lymphoma

A

Correct Answer: E.
This patient with IHIV/AIDS and a solitary expanding mass in the cerebral cortex most likely has primary central nervous system (CNS) lymphoma; a variety of non-Hodgkin lymphoma and an AIDS-defining malignancy. Primary CNS lymphoma classically presents with seizures, lethargy, subacute memory loss, and headache. Physical examination may disclose neurologic deficits stemming from structural disruption as a result of the location of the mass. Risk factors include a low CD4 count (often less than 50 cells/mm3} and a high HIV viral load. Clinical suspicion for CNS lymphoma should be highest in patients with uncontrolled HIV and a newly discovered brain mass. CNS lymphoma commonly involves the deep structures of the brain (eg. basal ganglia), cerebral white matter corpus callosum, and periventricular areas. Diagnosis is made by imaging, biopsy and cerebrospinal fluid analysis. The differential diagnosis for a space-occupying brain lesion in a patient with HIV/AIDS includes cerebral toxoplasmosis, brain abscesses, malignant primary brain tumors, and metastatic disease. MRI of the brain is helpful in differentiating between these diseases. Lumbar puncture is frequently performed as part of the evaluation for suspected primary CNS lymphoma unless there is a contraindication such as impending
brain herniation. Treatment includes methotrexate and highly active antiretroviral therapy. Rituximab, a monoclonal antibody against CD20, can be used as an adjunct.

Educational Objective: Solitary brain lesions in patients with HIV/AIDS, particularly when associated with a low CD4+ count and a high viral load, should raise suspicion for primary CNS lymphoma. Alternative diagnoses include a bacterial abscess, cerebral
toxoplasmosis, primary brain tumors, and metastatic lesions.

How well did you know this?
1
Not at all
2
3
4
5
Perfectly
21
Q

50-year-old man with type 2 diabetes mellrtus comes to the physician for a follow-up examination. Current medications include long-acting insulin and metformin. His vital signs are within normal limits. Physical examination shows no abnormalities. His
hemoglobin Alc is 7.9%. Treatment with a third drug that promotes release of endogenous insulin is initiated to improve glucose control. This drug is most likely which of the following?

A) Acarbose
B) Miglitol
C) Pioglitazone
D ) Pramlintide
E) Sitagliptin

A

Correct Answer: E.
Sitagliptin is an oral antihyperglycemic used in the control of blood glucose in type 2 diabetes mellitus. It is a competitive inhibitor of dipeptidyl peptidase-4 (DPP-4). DPP-4 is the enzyme responsible for breaking down gastrointestinal incretins; which are released
during a meal and promote the secretion of insulin. By preventing the breakdown of incretin hormones by inhibiting the enzyme responsible for degrading them, their effect (increasing endogenous release of insulin) persists. Examples of incretins include glucagonike
peptide-1 (GLP-1). Sitagliptin is generally a second-line medication, used when metformin, diet, exercise, and weight loss have failed to control blood glucose appropriately (as marked by a hemoglobin A-| C above 7.0%). In some cases, insulin ora sulfonylurea may be added prior to initiating a DPP-4 inhibitor.

Educational Objective: DPP-4 inhibitors prevent the early breakdown of incretins, which promote endogenous insulin release. Sitagliptin is an example of this class of inhibitor. They are generally used once metformin, diet, exercise, weight loss, or additiona medications have failed to appropriately control a patient’s blood glucose.

How well did you know this?
1
Not at all
2
3
4
5
Perfectly
22
Q

Lysosomes present in hepatocytes of patients with mucolipidosis II ( l-cell disease) contain abnormal enzyme activities resulting from an inability of these cells to do which of the following?
A) Glycosylate secretory proteins
B} Recycle lysosomes
C) Recycle mannose 6-phosphate receptors from the plasma membrane
D ) Synthesize lysosomal enzymes
E) Synthesize mannose 6-phosphate residues on proteins
-

A

Correct Answer: E.
Mucolipidosis II, also known as l-cell (inclusion cell) disease, results from accumulation of oligosaccharides, lipids, and glycosaminoglycans such as heparan sulfate and dermatan sulfate within cells, l-cell disease is a type of lysosomal storage disease. It is similar in phenotype and pathophysiology to Hurler syndrome, l-cell disease results from a failure of Golgi-based post-translational modification of proteolytic enzymes that would typically be targeted to the lysosome. Proteases targeted to the lysosome are tagged with
phosphate at the sixth carbon on their mannose residues, forming mannose 6-phosphate. Defective Golgi W-acetylglucosaminy1-1-phosphotransferase enzymes are unable to join phosphate onto mannose residues. This causes the synthesized proteases to be abnormally routed into vesicles for exocytosis instead of to the lysosome. In absence of these proteins within the lysosome, norma cellular debris that require lysosomal degradation accumulate within the cells and cause inclusions that can be seen on light microscopy. The resulting accumulation of such products leads to widespread cellular and organ dysfunction. Signs and symptoms of l-cell disease include failure to thrive, developmental delay, coarse facial features, restricted skeletal development,
hepatosplenomegaly, cardiac structural defects, corneal clouding, and dwarfism, l-cell disease demonstrates autosomal recessive inheritance; there is no treatment for the condition other than supportive care

Educational Objective: l-cell disease is an autosomal recessive lysosomal storage disease that results from a defect in W-acetylglucosaminyl-1-phosphotransferase enzymes. This results in the failure of phosphorylation of lysosomal hydrolases, which subsequently leads to their exocytosis from the cell instead of routing to their normal site of action within the lysosome.

How well did you know this?
1
Not at all
2
3
4
5
Perfectly
23
Q

A 30-year-old woman has polyhydramnios at 25 weeks’ gestation. The fetus has an anatomically normal esophagus. Which of the following abnormalities is the most likely cause of the polyhydramnios?
A) Anencephaly
B) Caudal dysplasia
C) Lumbar myelomeningocele
D) Renal agenesis
E} Urethral obstruction

A

Correct Answer: A.
Polyhydramnios describes the condition of excess aimniotic fluid within the amniotic sac during fetal development. It is normally detected on ultrasonography as increased fluid depth around the fetus. It can be caused by several known pathologies but is often diopathic. Anencephaly is a fetal neural tube defect in which the brain and cranial vault do not form correct y. It is a severe condition that leads to fetal loss or neonatal death. As the pregnancy progresses, anencephaly frequently results in polyhydramnios as the
malformations of the face and head do not allow the fetus to appropriately swallow amniotic fluid: thereby increasing the amount of fluid surrounding the fetus in the amniotic sac. Additionally, if the fetal central nervous system does not fully form in anencephaly. the stimulus to swallow along with appropriate neuromuscular pathways required to do so may be absent. An atretic esophagus can be a cause of polyhydramnios, also because of the inability to swallow and therefore clear amniotic fluid, however in this case, the
esophagus is normal.

Educational Objective: Amniotic fluid is recycled by fetal swallowing; malformations in the neurologic or craniofacial anatomy required to swallow (eg, anencephaly or esophageal atresia) may result in the accumulation of amniotic fluid, a condition known as polyhydramnios.

24
Q

A 45-year-old man has shortness of breath. He has never smoked cigarettes. There is no evidence of cardiac disease. X-rays of the chest show bullous air spaces in the lower lobes. The pathophysiologic basis of this process is most likely to be a deficiency of
which of the following?
A) Ceruloplasmin
B) Epidermal growth factor
C) Ferritin
D ) Plasminogen
E) Protease inhibitor

A

Correct Answer: E.
a1-Antitrypsin is a protease inhibitor that regulates neutrophil elastase. Neutrophil elastase is a serine protease secreted by neutrophils and macrophages as part of a localized inflammatory response. a1-Antitrypsin deficiency is an inherited disorder caused by mutations in the SERPtNA1 gene: which encodes -antitrypsin. Reduced concentration of effective protein result in host tissue damage from neutrophil elastase; especially in the lungs and liver. Pulmonary manifestations classically include panlobular emphysema which results in the development of bullous air spaces predominantly in the lower lobes. The diagnosis should be suspected in patients presenting with evidence of chronic lung disease with emphysematous changes and no history of tobacco use

Educational Objective: alpha-Antitrypsin deficiency should be suspected in patients who present with emphysematous lung changes in the absence of a smoking history, a^-Antitrypsin is a protease inhibitor responsible for regulating neutrophil elastase

25
Q

Which of the following curves best describes variation of follicle-stimulating hormone concentration during the menstrual cycle?

A

Correct Answer: B.
Follicle-stimulating hormone (FSH) is produced by the pituitary in response to gonadotropin releasing hormone from the hypothalamus. Its concentration fluctuates predictably with each menstrual cycle. At the start of the follicular phase, just after menses occurs,
FSH increases to support a developing ovarian follicle. Unlike the 14-day luteal phase, the length of the follicular phase is variable. As the follicle increases in size, it requires less support and FSH decreases slightly until ovulation occurs. Ovulation is
accompanied by a rapid surge in FSH and luteinizing hormone (LH): this marks the transition from the follicular phase to the luteal phase. FSH then rapidly subsides and increases slowly through the luteal phase as it prepares to support another ovarian follicle at
the start of the following cycle.
Incorrect Answers: A. C. D. and E.
LH (Choice A) remains low throughout the follicular phase until if surges and triggers ovulation. This marks the transition to the luteal phase, named for the presence of the corpus luteum. The corpus luteum is the structure that remains after a follicle ruptures and ovulation occurs. 11 produces estrogen and progesterone, which support the secretory endometrium and allow for implantation to occur. If implantation occurs, the corpus luteum is maintained by human chorionic gonadotropin and continues to produce
progesterone. If implantation does not occur, the carpus luteum shrinks and eventually becomes a fibrous remnant called the corpus albicans.
Progesterone (Choice C) is produced by the corpus luteum alter ovulation and maintains the secretory endometrium in order to allow for implantation to occur. If implantation does not occur, progesterone concentrations decrease and the endometrium beings to slough, causing menses. Estrogen (Choice D) is produced by the granulosa cells of the developing follicle. As the follicle increases in size through the follicular phase, its production of estrogen increases. A second, smaller peak of estrogen occurs in the luteal phase, which stimulates the endometrium to proliferate.
A hormone that fluctuates less during the overall course of the menstrual cycle is gonadotropin releasing hormone (GnRH). For example, Choice E may represent the pulsatile release of GnRH from the hypothalamus that occurs every 1 to 1 !4 hours in the
follicular phase and every two hours in the luteal phase. These small pulses stimulate the pituitary to secrete FSH and LH to drive the development of the ovarian follicle and ovulation. Educational Objective: FSH increases during the follicular phase of the menstrual cycle in order to support the developing ovarian follicle. It surges at the time of ovulafion. followed by a precipitous drop, and then subsequently slowly increases during the luteal phase, preparing to support the developing follicle of the next cycle.

26
Q

The breakdown of dipeptides and tripeptides to free amino acids takes place primarily in which of the following areas in the gastrointestinal tract?
A) Intestinal mucosa
B)} Lumen of the duodenum
C) Lumen of the large intestine
D ) Lumen of the stomach
E) Mouth

A

Correct Answer: A.

Small intestinal enterocytes within the brush border of the intestinal mucosa avidly absorb dipeptides and tripeptides through active transport across the luminal membrane. Once absorbed into the intestinal mucosa, dipeptides and tripeptides undergo further digestion to free amino acids within the cytoplasm. This process is mediated by cytoplasmic dipeptidases and tripeptidases. Free amino acids are secreted into the portal circulation for transport to the liver. Brush border peptidases, in addition to intracytoplasmic
peptidases: are integral to this process as only small peptides can be absorbed

Educational Objective: Dipeptides and tripeptides are produced by the activity of trypsin and chymotrypsin within the lumen of the duodenum. Free amino acids are produced upon further digestion of these small peptides within the brush border of the intestinal
mucosa.

27
Q

A 7-day-old female newborn is brought to the physician because blood has been oozing from the umbilical area for more than 24 hours. She was born at home, has had no previous medical treatment and has been breast-fed since birth. Which of the following biochemical reactions is most likely defective in this patient?
A) Cross-linking of collagen and elastin
B) Hydroxylation of proline
C) Post-translational carboxylation of glutamate residues
D) Synthesis of thymidylate

A

Correct Answer: C.
Vitamin K plays a critical role in the synthesis of hepatic coagulation proteins: it is oxidized in the liiver during the post-translational carboxylation of glutamate residues on coagulation factors ll; VII , IX . X; and proteins C and S. It is a co-factor for vitamin K epoxide reductase, which reduces vitamin K following its involvement in glutamate carboxylation. Vitamin K is often deficient in the neonatal period, as the sterile gastrointestinal tract of the neonate lacks the commensal bacteria that normally act as a source of vitamin K in the adult; additionally, the neonatal diet tends to be vitamin K poor, especially those who are breast-fed as it has a low concentration in breast milk. Newborns are often administered vitamin K at the time of birth to prevent bleeding in the neonatal period as a result of this natural deficiency. In this case, the home-born, breast-fed neonate is likely deficient in vitamin K, impairing the synthesis of coagulation proteins and therefore impairing hemostasis.
Educational Objective: Vitamin K deficiency is common in breast-fed, homeborn neonates since they lack commensal bacteria that would otherwise synthesize it in the gut. An absence of vitamin K leads to coagulopathy and bleeding since factors II, VII , IX . and X require vitamin K for the post-translational carboxylation of glutamate residues during synthesis.

28
Q

A 45-year-old man is brought to the physician by his parents because he is reclusive, has “weird ideas,” and still wants to live at home. He is dressed in several layers of clothes. During the interview; he is cooperative but he has poor eye contact and seems uninterested in the interview. He says; “I believe some people can read other people’s minds. I carry these crystals to help me realize self-actualization.” He has no delusions or hallucinations. This patient most likely has which of the following personality disorders?
A) Borderline
B) Dependent
C) Histrionic
D) Narcissistic
E) Schizoid
F) Schizotypal

A

Correct Answer: F.
Schizotypal personality disorder is one of the cluster A personality disorders, the odd or eccentric cluster. The disorder is characterized by odd behavior and thinking and a constricted affect. Patients with schizotypal personality disorder may have strange, overly metaphorical or magical thinking, though they fluctuate in the degree of their conviction about these beliefs (which differentiates these beliefs from the fixed delusions of schizophren a). The strange beliefs may also manifest as an eccentric appearance as in this patient. Patients may experience somatosensory illusions or other abnormal perceptions. Patients with schizotypal personality disorder may appear to lack interest in cultivating relationships and, because of an inability to interpret others7 motivations, may be deeply distrustful and anxious around others. Personality disorders are persistently maladaptive ways of relating to the self and to society that typically appear by early adulthood.

Educational Objective: Schizotypal personality disorder manifests as an enduring pattern of strange or eccentric beliefs and behavior, social detachment or suspiciousness, and abnormal perceptual experiences such as illusions. Social indifference distinguishes schizotypal disorder from cluster B and C disorders, and eccentric beliefs and behavior differentiate the disorder from the other cluster A disorders.

29
Q

A 42-year-old man conies to the physician because of chronic abdominal pain for 3 months. He describes his symptom as boring epigastric pain that is occasionally exacerbated by eating. He is an investment banker and describes his job as extremely
stressful. Use of an over-the-counter histaminergic (HJ-receptor antagonist has mixed results. Vital signs are normal. Physical examination and upper endoscopy show no abnormalities. Which of the following is the most likely diagnosis?
A) Gastric lymphoma
B) Generalized anxiety disorder
C) Irritable bowel syndrome
D) Major depressive disorder
E) Nonulcer dyspepsia

A

Correct Answer: E.
Nonulcer dyspepsia is the most likely diagnosis in this patient with epigastric pain, work-related stress, and normal endoscopic evaluation. Dyspepsia is a term used to describe epigastric pain variably accompanied by multiple symptoms including nausea; bloating, or early satiety. The most common causes of dyspepsia include peptic ulcer disease (PUD) and gastroesophageal reflux disease (GERD). While guidelines vary: most patients over the age of 60, or those with symptoms concerning for a possible malignancy such as weight loss or dysphag a: should undergo esophagogastroduodenoscopy (EGD) to rule ouf gastric or esophageal cancer. In younger patients, or those without red-flag features, a trial of an H2-receptor antagonist or proton pump inhibitor (PPI) is considered firstine therapy. Failure to respond to appropriate medical therapy should prompt additional evaluation. Patients should undergo testing for Helicobacter pylon with stool antigen testing (if not taking a PPI) or urea breath test. Positive tests should be followed by appropriate therapy to eradicate H pylori. Patients who have normal evaluations with persistent symptoms are diagnosed with nonulcer (functional) dyspepsia. These patients commonly have comorbid psychiatric conditions including generalized anxiety disorder or major depressive disorder, and stressful life circumstances are supportive of this diagnosis.

Educational Objective: Nonulcer dyspepsia, also known as functional dyspepsia, is a cause of epigastric pain that is common in patients with other somatic symptoms and increased levels of stress. Diagnosis is made after thorough evaluation to exclude organic
causes of dyspepsia.

30
Q

A 25-year-old woman comes to the physician 2 days after noticing a mass in her right axilla. She also has a 1-week history of malaise, headaches, and night sweats. The patient says that she recently adopted a kitten and has sustained several bite and
scratch marks. Her temperature is 37.8aC (1GQCF). Physical examination shows edema and tenderness of the right axillary lymph node. The skin over the node is erythematous, tough, and warm. There are scratches and bite marks of various ages over the
upper extremities and hands. The result of an indirect fluorescent antibody test for Bartonella henselae is positive. A biopsy specimen of this lymph node is most likely to show which of the following histologic patterns?
A ) Diffuse neutrophil infiltration
B) Granulomas containing stellate microabscesses
C) Large activated lymphocytes with occasional plasma cells
D ) Serous inflammation with abundant intravascular fibrin and few inflammatory cells
E) Vascular proliferation with fibrosis

A

Correct Answer: B.
Cat-scratch disease is caused by infection with the gram-negative coccobacillus Bartonella henselae. Infection is acquired via scratches or bites from domestic or feral cafs: which may be localized or disseminated, but most commonly causes local lymphadenitis in the lymphatic drainage pattern of the scratch location. The most involved lymph nodes are the axillary and cervical lymph nodes. Histologic examination typically demonstrates necrotizing granulomas with stellate (star-shaped) microabscesses. Multinucleated giant
cells may or may not be present

Educational Objective: Cat-scratch disease is caused by infection with the gram-negative coccobacillus Bartonella henselae. Histologic examination of involved lymph nodes typically demonstrates necrotizing granulomas with stellate (star-shaped) microabscesses.
Multinucleated giant cells may or may not be present.

31
Q

A 3-year-old boy is brought to the emergency department by his mother 1 hour after having a generalized tonic-clonic seizure. He has had frequent vomiting and progressive stupor during the past 24 hours. His mother says that she gave him aspirin 2 days ago because of a temperature of 39.5X (103.1aF). Physical examination shows hepatomegaly with no jaundice. Serum studies show an increased ammonia concentration. and increased AST and ALT activities. Dysfunction of which of the following is the
most likely cause of this patient’s disorder?
A ) Glutamate transporters
B) Golgi complex
C) Microtubules
D ) Mitochondria
E) Rough endoplasmic reticulum

A

Correct Answer: D.
Mitochondrial dysfunction is associated with the pathogenesis of IReye syndrome; a rare, frequently fatal encephalopathy resulting from liver dysfunction in children. Reye syndrome presents in stages one through five with early stages demonstrating nonspecific rash; lethargy, confusion, nausea and vomiting, eventually leading to stupor. In later stages; coma; cerebral edema; seizures, organ failure, hyperammonemia, and death can result. Physical examination typically shows depressed mental status or loss of consciousness depending on the stage as well as hepatomegaly in later stages. Reye syndrome classically presents in children and is rare in adults; it may be idiopathic but is associated with salicylate use (eg: aspirin) during a preceding viral infection. Influenza viruses, especially influenza B are most frequently implicated, though varicella has also been described as a precipitant. Patients with genetic deficiencies of fatty acid oxidation (eg, medium-chain acyl-CoA dehydrogenase deficiency) appear to be at higher risk.
Salicylate metabolites are proposed to disrupt p-oxidation of fatty acids within mitochondria leading to fatty liver and the progressive hepatic dysfunction that characterizes the syndrome (hepatic encephalopathy, hyperammonemia, increased serum transaminases). Prevention is key. The only generally accepted use of salicylates in children is for mucocutaneous lymph node syndrome (Kawasaki disease). Otherwise, the use of salicylates should be avoided because of the risk for Reye syndrome. Treatment is supportive, including occasional use of mannitol to decrease cerebral edema. Even with treatment, Reye syndrome is often fatal.

Educational Objective: Mitochondrial dysfunction is associated with the pathogenesis of Reye syndrome, which is a rare, frequently fatal encephalopathy resulting from liver dysfunction in children. It is most associated with salicylate use (eg. aspirin) in children during a preceding viral infection.

MitochonDREYEa, Golgi-Cell

32
Q

42-year-old man conies to the physician for an examination prior to beginning an exercise program at a local health club. He has smoked 1 pack of cigarettes daily for 20 years. He is 183 cm (6 ft} tall and weighs 70 kg (155 lb); BMI is 21 kg/m2 His blood pressure is 130/S0 mm Hg. Physical examination shows no other abnormalities. Serum studies show:
Cholesterol total 350 mg/dL
HDL-cholesterol 40 mg/dL
LDL-cholesterol 280 mg/dL
Triglycerides 150 mg/dL

Which of the following defects is the most likely cause of this patient’s increased serum LDL-cholesterol concentration?

A ) Decreased ability to convert cholesterol to bile acids
B) Decreased clearance of cholesterol by the liver
C) Increased absorption of cholesterol in the gut
D ) Increased production of cholesterol by adipose tissue
E) Increased production of cholesterol by the liver

A

Correct Answer: B.
Famil al hypercholesterolemia (also known as type II familial dyslipidem a) is a relatively common, autosomal dominant disorder of cholesterol metabolism. The disorder is caused by mutations in the LDLR; PCSK9, or APOB genes. Defects in these genes lead to decreased clearance of cholesterol by the liver and increased serum cholesterol concentrations. LDLR encodes the LDL receptor (LDILR) expressed on hepatocytes, and loss of function mutations decrease LDL uptake by the liver. Gain-of-function mutations in PCSK9 lead to decreased LDLR expression through formation of the PCSK9-LDLR complex, which is internalized by the cell and unable to be recycled to the cell surface. Apolipoprotein B-100 (ApoB-100) defects reduce the ability of the LDL receptor to bind LDL. The diagnosis is suspected when increased cholesterol concentrations are discovered on routine laboratory testing in the absence of secondary causes for hypercholesterolemia such as alcohol use, d abetes mellitus, hypothyroidism, polycystic ovary syndrome, glycogen LDLR storage diseases, nephrotic syndrome, and cholestatic liver disease. It should also be considered when patients present with severe atherosclerotic disease early in life or have a first-degree relative with history of the same. In addition to statin therapy, the patient should be counseled on lifestyle modifications to reduce the lifetime riiskfor premature coronary artery disease, such as the cessation of tobacco use.

Educational Objective: Familial hypercholesterolemia is an inherited disorder leading to increased concentrations of cholesterol in the blood and a major risk factor for premature coronary artery disease and death if untreated. The primary mechanism of serum cholesterol accumulation is decreased clearance by the liver.

33
Q

The formula shown describes the rate of metabolism of Drug X where v is the rate of metabolism,
concentration of Drug X is 100 times greater than the Kmof the metabolic enzyme for Drug X. which of the following conclusions is most appropriate?
A ) First-order kinetics governs the rate of the reaction
B) Further metabolism of Drug X is inhibited
O C) Metabolism of Drug X involves covalent bonding of the enzyme
D ) The velocity of the reaction is proportional to the concentration of Drug X
E } A zero-order rate of metabolism governs the reaction

A

Correct Answer: E.

Educational Objective: At high concentrations of Drug X, reactions which follow
Michaelis-Menten pharmacokinetics will demonstrate zero-order kinetics because of enzymatic saturation. The maximal rate of reaction in a completely saturated environment will remain constant, independent of further changes in concentration.

34
Q

An investigator studying type 2 diabetes discovers an inhibitor of uncoupling protein-2 that stimulates insulin secretion from mouse pancreatic is et cells. This inhibitor most likely increases insulin secretion by which of the following actions?
A ) Inactivating a tyrosine kinase
B) Increasing glucokinase activity
C) Increasing glucose transporter-2 (GLUT-2) expression
D ) Increasing the ATP:ADIP ratio
E) Inhibiting the target of rapamycin (TOR) protein kinase
F ) Stimulating the hydrolysis of phosphoinositide

A

Correct Answer: D

Insulin secretion takes place over several steps. As glucose is transported into the islet cell by the GLUT-2 transporter, glycolysis occurs, which increases the ratio ofATP:ADP. In turn, ATP binds to the ATP-sensitive potassium channels, which close, leading to a change in the transmembrane potential. The cell depolarizes, and calcium ions accumulate within the cytoplasm, which trigger exocytosis of pre-formed insulin. Uncoupling proteins are found in mitochondria, and in this setting, permit dissipation of energy in the electron transport chain as heat, instead of shunting all protons into a gradient and all electrons down the cytochromes to produce ATP. Uncoupling proteins in this manner permit disruption of the proton gradient, uncoupling the gradient from ATP synthesis. If an uncoupling protein were therefore inhibited, as in this experiment, any dissipation of the proton gradient over such a protein would be limited, and al of the stored energy would be converted to ATP, thus, raising the ATP:ADP ratio within the cell.

Educational Objective: Increases in the ratio of ATP:ADP lead to islet cell depolarization by closing potassium-ATP channels. Calcium ions enter the cell, triggering the exocytosis of pre-formed insulin. Uncoupling protein prevents some of the proton gradient in a mitochondrion from being used in the synthesis of ATP. Blocking such a protein would theoretically increase the cell’s ATP:ADP ratio.

35
Q

An investigator is studying a tumor cell line in an experimental animal model in which the expression of class I MHC is completely inhibited. The cell line is assayed as a target for cell-mediated immune response by CD3+ T lymphocytes, CD4+ T lymphocytes; or natural killer cells. Which of the following sets of findings is most likely (+ = killing: - = no killing}?

CDS+ T Lymphocytes CD4+ T Lymphocytes Natural Killer Cells

A

Educational Objective: Natural killer cells use perforin and granzyme to induce apoptosis in virally infected or neoplastic cells that do not express MHC class I antigens. NK cells respond to the absence of MHC class I.

*The question is asking what responds to missing class I, not what can function when class I isnt present, which is why CD4+ cells are not correct. *

36
Q

A 23-year-old woman, gravida 4, para 0. aborts 3, with systemic lupus erythematosus comes to the physician at 12 weeks’ gestation for an initial prenatal examination. She is concerned that she will not be able to carry her current pregnancy to term. The presence of which of the following is most likely increasing the risk for complications during pregnancy in this patient?
A) Antimitochondrial antibodies
B) Antineutrophil cytoplasmic antibodies
C) Antiphospholipid antibodies
D) Antistreptolysin O antibodies
E) Rheumatoid factor

A

Correct Answer: C.
Antiphospholipid syndrome (APLS); which is an autoimmune disorder that can be primary or secondary to an additional autoimmune disorder (eg: systemic lupus erythematosus), often presents with recurrent miscarriages, along with arterial or venous thromboembolic disease. Diagnosis of APLS requires the presence of a clinical event such as thrombosis or stillbirth, along with blood tests separated by at least three months that detect antiphospholipid antibodies including anticardiiolipin antibodies, antiapolipoprotein antibodies, or lupus anticoagulant factor. Pregnancy-related complications involving the fetus include stillbirth, intrauterine growth restriction, and preterm labor. Maternal complications are frequent and include deep venous thrombosis, or arterial or venous blood clots in nearly any organ. Strokes and pulmonary emboli are common in pregnant women with APLS. Clots involving the placenta may lead to infarction, which is one hypothesized mechanism for the pathogenicity related to the fetus. The antibodies that are produced in APLS often bind to or interfere with the action of proteins C and S, shifting the balance of hemostasis in favor of hypercoagulability.

Educational Objective: Antiphospholipid syndrome, an autoimmune disorder that can be primary or secondary to an additional autoimmune disorder (eg, systemic lupus erythematosus), often presents with recurrent miscarriages along with arter al or venous thromboembolic disease. The presence of antiphospholipid antibodies confirms the diagnosis when paired with a known, associated clinical event.

37
Q

A 55-year-old woman comes to the physician because of a 3-day history of persistent right shoulder pain. She began a weight-training program 6 weeks ago. She says that the pain intensified after she increased the amount of weight that she had been lifting
above her head. She rates the pain as 8 on a 10-point scale. Examination of the right shoulder shows point tenderness just lateral to the acromion, over the humeral head. Passive motion of the shoulder is full. Pain is reproduced with resisted abduction of the
shoulder when the shoulder is abducted 90 degrees and the arm is giving the “thumbs down” sign. Sensation is intact over the right upper extremity. Which of the following tendons is most likely injured in this patient?
A ) Deltoid
B) Infraspinatus
C) Subscapularis
D ) Supraspinatus
E) Teres major
F ) Teres minor

A

Correct Answer: D.
The supraspinatus tendon is commonly injured or impinged during overhead motion, especially in persons unaccustomed to such motion. It is one of four tendons associated with the rotator cuff muscle group and is involved in glenohumeral joint stabilization as with the remainder of this muscle group. It is the continuation of the supraspinatus muscle, and it courses beneath the acromion process and superior to the humeral head to insert on the greater tuberosity of the humerus. The muscle originates in the supraspinous fossa of the scapula. As the tendon courses inferior to the acromion, it can become impinged or inflamed. As the primary role of the supraspinatus is the initiation of shoulder abduction (first 15 degrees), patients may report pain with abduction or inability to abduct the shoulder, and the pain may be worsened with maneuvers that exacerbate tendon impingement, such as resisted abduction beyond 90 degrees with internal (medial) rotation of the extremity. Diagnosis is generally based on the clinical examination and association of symptoms with maneuvers known to provoke symptoms; treatment involves rest, nonsteroidal anti-inflammatory drugs, and physical therapy with range-of-motion exercises. Other muscles involved with the rotator cuff are the infraspinatus, feres minor, and subscapularis.

Educational Objective: The supraspinatus tendon is commonly injured or impinged during overhead motion. Abduction with internal rotation of the arm can produce pain with impingement of the injured tendon.

38
Q

A 37-year-old woman comes to the physician for a follow-up examination 3 months after receiving a kidney transplant from a cadaver donor. She feels well and her condition has been stable since the transplant. She receives standard immunosuppressive medications Her temperature is 37°C (98.6aF)? pulse is 347min respirations are 14/min. and blood pressure is 150/90 mm Hg. Physical examination shows a well-healed surgical scar over the lower abdomen. Serum studies show a creatinine concentration of 2 mg/dL; baseline creatinine concentration was 1.4 mg/dL 1 week post transplant and 2 weeks ago. Serum concentrations of the immunosuppressive drugs are in the low therapeutic range. Examination of a renal biopsy specimen shows lymphocytes in the
tubules and arterial walls. Which of the following best explains these findings?
O A) Acute antibody-mediated rejection
B) AcuteT-lymphocyte-mediated rejection
C) Adverse effects of immunosuppressant medications
D ) Chronic allograft rejection
E) Hyperacute rejection

A

Correct Answer: B.
Acute T-lymphocyte-mediated rejection (TCMR) is characterized by transplant allograft organ dysfunction (eg, increased creatinine), and transplant organ histology (in the case of the kidney) demonstrating lymphocytes in the tubules and arterial walls, often with
inadequate immunosuppressive drug concentrations. This most commonly occurs within the first 6 months after transplant. TCMR is caused by host T-lymphocytes reacting to donor major histocompatibility complexes (MHC) present on cells within the glomeruli,
tubules, interstitium. and blood vessels of the allograft. Biopsy is required for the evaluation of graft dysfunction because it can distinguish between acute rejection and other causes of kidney injury, including acute tubular necrosis, interstitial nephritis, or viral
infections (eg, cytomegalovirus or BK virus). Primary histologic changes include interstitial infiltration with lymphocytic cells, in addition to obliteration of the tubular basement membrane. Treatment is based upon the degree of rejection and allograft dysfunction but includes steroids and lymphocyte-depleting agents.

Educational Objective: Acute T-lymphocyte-mediated rejection (TCMR) typically occurs within months following transplantation and presents with allograft dysfunction. Kidney biopsy demonstrates lymphocytes in the interstitium, tubules, glomeruli, and blood vessels

39
Q

A 45-year-old woman with chronic hepatitis C comes to the physician because of a 3-week history of fatigue and joint pain. Her temperature is 37.5°C (99.5°F); pulse is 88/min, and blood pressure is 155/75 mm Hg. Physical examination shows the findings in the photograph and hepatosplenomegaly. Serum studies are most likely to show an increased amount of which of the following in this patient?
A) Cholesterol
B) Complement
C) Creatine kinase
D) Cryoglobulin
E) D-Dimer

A

Correct Answer: D.
Cryoglobulins are immunoglobulins (or immunoglobulin-complement complexes) that precipitate out of serum. Risk factors for excessive cryoglobulin formation include chronic viral infections such as hepatitis B and C viruses, human immunodeficiency virus (HIV), malaria, and Ebstein-Barr virus (EBV), chronic inflammatory conditions such as systemic lupus erythematosus and Sjogren syndrome, and IymphoproIiterative diseases such as multiple myeloma. The deposition of the immunoglobulin-complement complexes in blood vessels results in leukocytoclastic vasculitis, which can manifest with a reddish-purple rash (purpura) and possible end-organ dysfunction. Mixed cryoglobulinemia is characterized by the triad of weakness, arthralgias, and palpable purpura. Patients may also present with peripheral neuropathy, hematuria, and hepatosplenomegaly.

Educational Objective: Increased cryoglobulin concentrations are characteristic of mixed cryoglobulinemia, which often presents with palpable purpura in association with arthralgias and peripheral neuropathy caused by immune-complex mediated vasculitis. Chronic hepatitis C virus is the most common infection associated with cryoglobulinemia.

40
Q

A 30-year-old man comes to the office because of a 1-week history of nausea and yellowing of the whites of his eyes. He has no history of serious illness and takes no medications. Vital signs are within normal limits. Abdominal examination shows hepatomegaly; there is tenderness to palpation over the right upper quadrant Serologic testing is positive for hepatitis A virus IgM antibodies. A high-power photomicrograph of a liver biopsy specimen obtained from a patient with a similar condition is shown. Which of the following is the most likely mechanism of hepatocyte cell death in this patient?
A) Apoptosis caused by activation of the death receptor extrinsic pathway
B) Apoptosis caused by activation of the mitochondrial intrinsic pathway
O G) Autophagy caused by inflammation
D ) Necrosis caused by free radical injury
E) Necrosis caused by lack of blood supply

A

Correct Answer: A.
The liver photomicrograph of this patient with acute viral hepatitis A infection demonstrates ballooning degeneration of hepatocytes and an intensely eosinophilic apoptotic body (Councilman body). Apoptotic bodies represent hepatocytes in a state of apoptosis or necrosis, and their distinct appearance histologically often reflects contrast from surrounding normal or ballooning hepatocytes. Apoptotic bodies are most frequently observed in viral hepatitis caused by hepatitis A. although they may also be observed in association with yellow fever. Viral hepatitis A infection leads to hepatocyte death primarily through apoptosis caused by activation of the death receptor extrinsic pathway. While the intrinsic and extrinsic pathways of apoptosis both converge on caspase-mediated cell death: the extrinsic pathway is dependent on external cellular signals as with the binding of Fas and Fas-ligand or. in this case:i the release of perforin and granzyme B from cytotoxic CD8+ T Lymphocytes responding to intracellular viral pathogens. Lymphocytic infiltrates are frequently seen on liver histology in the setting of an acute hepatitis infection.

Educational Objective: Acute hepatic inflammation in the setting of viral hepatitis A or yellow fever may cause apoptosis of hepatocytes. This occurs via the extrinsic pathway of apoptosis and is seen on histology as ballooning hepatocytes and apoptotic bodies.

41
Q

A 7-year-old girl is admitted to the hospital because of severe anemia. Hemolytic anemia with a probable defect in glycolytic energy production is diagnosed. Results of pyruvate kinase activity in erythrocyte lysates using varying concentrations of phosphoenolpyruvate are shown. Which of the following best explains these findings?

A) Decrease in the rate of enzyme synthesis
B) Increase in the rate of enzyme degradation o
C) Mutation in the enzyme that decreases Vmax
D ) Mutation in the enzyme that increases Km
E) Presence of a noncompetitive inhibitor in cells
F ) Presence of an irreversible inhibitor in cells

A

Correct Answer: D.

Educational Objective: Enzymatic mutations may alter the affinity or velocity of an enzyme for its substrate,marked by deviation from normal in Km or Vmax - A higher Km indicates decreased affinity of the enzyme active site for the substrate.

42
Q

An experiment is designed to assay the secretions of chondrocytes obtained from the articular cartilage of osteoarthritic joints. Which of the following compounds is most likely to be found in decreased concentration in the secretions?
A) Catabolic metalloproteinase
B) lnterleukin-1 (IL- 1)
C) InterleuKcin-G (IL-6)
D) Proteoglycans
E) Tumor necrosis factor

A

Correct Answer: D

Educational Objective: Proteoglycans comprise the extracellular matrix and are a key element of articular cartilage. In disease states where cartilage breaks down, such as osteoarthritis, decreased proteoglycan secretion would be expected.

43
Q

A healthy 40-year-old man is participating in a study in which he is given an intravenous injection of insulin to induce hypoglycemia. The graph shows serum concentrations of catecholamines at baseline and the 2500 -
peak values at the nadir of serum glucose. Which of the following most likely underlies the different magnitude of the epinephrine and norepinephrine responses?
A) Activation of catechol-O-methyltransferase
B) Failure of monoamine oxidase to metabolize epinephrine
C) Increased stability of circulating epinephrine
D) Reuptake of norepinephrine into sympathetic synapses
E) Selective activation of medullary epinephrine release

A

Correct Answer: E.

Educational Objective: Epinephrine plays a critical role in glucose homeostasis through its action on adrenergic receptors located on the liver, the activation of which results in glycogenolysis and gluconeogenesis. The selective activation of medullary epinephrine release may reflect the critical need for epinephrine to serve this function during times of hypoglycemia

44
Q

An analysis of patients admitted to a specific hospital finds that the risk for myocardial infarction is higher in men with chronic obstructive pulmonary disease (CORD) than in men without CORD (odds ratio 5.4, 95% confidence interval: 1.4-10.1). These results
can most reasonably be generalized to which of the following populations?
A ) Men with CORD and myocardial infarction
B) Men with CORD in the hospital
C) Men with COPD living at home
D ) Men with myocardial infarction in the hospital
E) All patients with COPD and myocardial infarction
F ) All patients with COPD in the hospital
G) All patients with COPD living at home
H ) All patients with myocardial infarction

A

Correct Answer: B.

Educational Objective: A study is generalizable to patients that are similar to the subjects studied. While results may be used to guide treatment in these cases, they are not necessarily generalizable to patients who do not meet the same demographic criteria as the
subjects enrolled in the study.

45
Q

A 34-year-old man is brought to the emergency department semiconscious and combative. In addition to sedation, a short-acting neuromuscular blocking agent is administered for intubation to prevent aspiration. Within a few seconds after administration of the drug, he has transient muscle fasciculations in his face: he develops generalized paralysis within 1 minute. Forty-five minutes after completion of the procedure, he is still paralyzed. A genetic abnormality of which of the following enzymes is the most likely cause of his unusually slow recovery from paralysis?
A ) Angiotensin-converting enzyme
B) Choline O-acetyltransferase
C) Monoamine oxidase
D ) Ptienylethanolamine A/-methyltransferase
E) Pseudocholinesterase
F ) Tyrosine hydroxylase

A

Correct Answer: E.

Educational Objective: Succinylcholine is a depolarizing neuromuscular blocker. It causes depolarization, seen clinically as fasciculations, followed by skeletal muscle relaxation. It is degraded by pseudocholinesterase, and its action is prolonged by a deficiency of this enzyme.

46
Q

A 50-year-old woman comes to the physician because of progressive shortness of breath during the past 2 years. Her respirations are 20/min. Physical examination shows cyanosis and ankle edema. Her pulmonary artery pressure, pulmonary vascular resistance, and right atrial pressure are increased: her pulmonary capillary wedge pressure is 0.3 mm Hg {N=S—16). Her ventilation improves when inhaled nitric oxide is administered. Treatment with oral bosentan is begun for long-term therapy. Which of the following mediators of pulmonary resistance in this patient will most likely be antagonized by this drug?
A ) Adenosine
B) Calcitonin gene-related peptide
C) C-reactive protein
D) Endothelin
E) Prostacyclin (PGIJ

A

Correct Answer: D.

Bosentan is a competitive endothelin-1 antagonist that is used for the treatment of primary pulmonary arterial hypertension. Two clinically relevant subtypes of endothelin receptors are the endothelin A (ETA) and the endothelin B (ETB) receptors. Binding of endothelin-1 to ETA results in vasoconstriction, whereas binding of endothelin-1 to ETB results in production of nitric oxide and vasodilation. In the context of pulmonary arterial hypertension, the aggregate effect of endothelin-1 results in vasoconstriction and an increase in pulmonary vascular resistance. Bosentan competitively antagonizes the binding of endothelin-1 to both ETA and ETB receptors. However, its greater affinity for ETA results in a vasodilatory effect and decreased pulmonary vascular resistance, limiting pulmonary hypertension, and relieving strain on the right side of the heart. Bosentan is hepatotoxic: patients treated with bosentan should be monitored for elevations of serum transaminases.

Educational Objective: Bosentan is a competitive antagonist of endothelin-1 and causes reduced pulmonary vascular resistance. It is effective in the treatment of pulmonary arterial hypertension.

47
Q

A 69-year-old woman comes to the physician because she has had weakness of her left leg since awakening that morning. Physical examination shows weakness of the extremity. A Babinski sign is present on the left. Sensory testing shows decreased somatic sensation in the left foot, agraphesthesia on the plantar surfaces of the toes; and decreased position sense in the toes. An MRI of the brain shows an edematous area in the cerebral cortex of the right hemisphere. The most likely cause of this condition is a lesion located at which of the following labeled areas in the photographs of normal brains shown?

A

Correct Answer: G.
Educational Objective: The lower extremity is controlled by the medial aspects of the precentral (motor) and postcentral (sensory) gyri in the region referred to as the paracentral lobule. Strokes affecting this region result in the loss of motor and sensory function of the contralateral lower extremity

48
Q

A 55-year-old woman is brought to the emergency department by police 1 hour after she was found wandering around the city park at midnight, appearing confused. She told the police that she is a prominent physician and a member of the city council. She is unwashed,, malodorous. and disheveled. She is 160 cm (5 ft 3 in) tall and weighs 55 kg (121 lb): BMI is 21 kg/m2 Her pulse is 82/min, and blood pressure is 110/82 mm Hg. Physical examination shows a broad-based gait and nystagmus. She is not oriented to person, place, or time, but she is pleasant and cooperative. Results of alcohol and drug screening are negative. An MRI of the brain is most likely to show atrophy of which of the following?
A ) Amygdala
B) Hippocampus
C) Hypothalamus
D ) Mammillary bodies
E) Parietal lobe

A

Correct Answer: D.
Educational Objective: Chronic alcohol use disorder and associated malnutrition may result in significant vitamin B1 (thiamine) deficiency, which can cause Wernicke-Korsakoff syndrome. The syndrome is characterized by symptoms such as altered mental status, ophthalmoplegia, ataxia, and retrograde and anterograde amnesia. The mammillary bodies are classically atrophied in Wernicke-Korsakoff syndrome

49
Q

During a study of familial hypercholesterolemia, hepatocytes from an affected patient are extracted and analyzed. Results show that the LDL receptors are evenly distributed on the surface of the cells. The cells also bind LDL-cholesterol with normal affinity but cannot internalize it. The cells are shown to internalize transferrin. Based on these findings, which of the fallowing is the most likely location of the causal mutation in this patient?
A ) Apo B
B) Apo E
C) Clathrin heavy chain
D ) Clathrin light chain
E) Cytoplasmic domain of the LDL receptor
F) Extracellular domain of the LDL receptor

A

Correct Answer: E.

Educational Objective: The cytoplasmic domain of the LDL receptor is responsible for signaling that leads to endocytosis, so mutations at this location allow for normal binding of LDL but an inability to transport it across the membrane. Mutations that cause dysfunction or absence of the LDL receptor result in familial hypercholesterolemia

50
Q

An 18-year-old man with Crohn disease is admitted to the hospital because of a 1-day history of severe abdominal pain and intermittent bloody diarrhea. His temperature is 38°C (100 4DF), pulse is 98/rnin. and respirations are 18/niin Physical examination shows a draining anal fistula. Treatment with broad-spectrum antibiotics and prednisone, along with intravenous hydration and parenteral nutrition, is initiated. He recovers over the next 3 weeks. In addition to resolving the infection, the most likely mechanism of action of this pharmacotherapy is suppression of which of the following?
A ) Antibody binding
6 } Complement activity
C) Mast cell degranulation
D) Neutrophil function
E) T-lymphocyte function

A

Correct Answer: E.

Educational Objective: Glucocorticoids such as prednisone primarily exert their anti-inflammatory properties by inhibiting the production and function ofT lymphocytes. Prolonged use has myr ad consequences including impaired lipid metabolism, insulin resistance, and deleterious effects on bone health.

51
Q

A previously healthy 18-year-old woman conies to the physician because of a 3-day history of pain with urination and urinary frequency and urgency. Physical examination shows mild tenderness over the suprapubic region. A urine culture grows an organism that is lactose fermenting and spot-indole test positive. The organism is resistant to ampicillin but sensitive to ceftriaxone. Which of the following is the most likely mechanism of antimicrobial resistance exhibited by this organism?
A ) Alteration of the existing penicillin-binding protein
6 } Changes iin porin
C) Efflux pumps
D ) Elaboration of a new penicillin-binding protein
E) p-Lactamase production

A

Correct Answer: E.
Educational Objective: Gram-negative and gram-positive bacteria have different modes of acquiring antibiotic resistance that lead to differing clinical patterns in virulence, p-lactamases inactivate p-lactam antibiotics and are a common mode of resistance to the plactam
class among gram-negative bacteria.

52
Q

A 57-year-old woman comes to the physician 3 days after she found a lump in her left breast on self-examination. Physical examination shows a 1,3-cm mass in the left breast. Microscopic examination of a biopsy specimen of the mass shows carcinoma. It is most appropriate for the physician to consider which of the following when grading the carcinoma?
A ) Distant metastasis
6 } Maximum tumor size
C) Number of mitoses per unit area
D ) Peritumoral lymphocytic response
E) Presence of extracellular mucin
F) Regional lymph node metastasis

A

Correct Answer: C.
Tumors are classified by grade and stage. Grading describes the cellular and histologic features of the tumor. Low grade tumors remain well-different ated and have a better prognosis than high grade tumors: which are poorly differentiated or undifferentiated.
Grading systems differ by the type of tumor being described. A high number of mitoses per unit area is evidence of rapid cell growth and division and is a common feature of high-grade tumors. In general, the stage of a tumor is more prognostically significant than grade. Staging describes the size and extent of a primary tumor the degree of involvement of regional lymph nodes: and the presence of distant metastases. 11 is usually standardized using the TIMM* (Tumor Nodes: Metastasis) staging system.

Educational Objective: Tumor grading describes the cellular and histologic features of a tumor. The number of mitoses per unit area; when increased: reflects active cell growth and division within a tumor

53
Q
A

Correct Answer: D.
Rheumatoid arthritis is an autoimmune disease that causes extensive inflammation in synovial tissues. The clinical course is characterized by joint destruction, periarticular bone loss; and synovial hypertrophy, along with extra-articular manifestations such as rheumatoid nodules and skin rash. Patients often present with symmetric patterns of arthritis particularly in the wrists and hands in middle-aged women. Serologic studies generally demonstrate a positive rheumatoid factor, and anti-cyclic citrullinated peptide antibodies. Rheumatoid arthritis is associated with the HLA-DR4 genotype.

54
Q

A 65-year-old woman comes to the emergency department because of a 4-hour history of vomiting bright red blood; she also has had dizziness and nausea during this period. Her pulse is 140/min and blood pressure is 70/30 mm Hg. Physical examination shows pale: clammy skin: diaphoresis, and decreased capillary refill time. A diagnosis of hypovolemic shock is made. Placement of a catheter in the internal jugular vein for blood transfusions is planned. Improper insertion of the catheter is most likely to result in damage to which of the following sets of underlying structures?
A ) Common carotid artery and pulmonary artery
6 } Esophagus and common carotid artery
C) Esophagus and pulmonary artery
D ) Lung and common carotid artery
E) Lung and esophagus

A

Educational Objective: Central venous catheter insertion is indicated for volume resuscitation, emergency vascular access, administration of vasopressors or other caustic medications, and facilitates interventions such as plasmapheresis, hemodialysis, and intracardiac pacing. A potential site of central catheter insertion, the internal jugular vein, is adjacent to the lung and common carotid artery, and complications from puncturing or injuring these structures include pneumothorax and hematoma or hemorrhage,
respectively.

55
Q

A 57-year-ald woman comes to the emergency department because of a 7-week history of fever night sweats, and nonproductive cough; she also has had a 6.8-kg (15-lb) weight loss during this period. She is currently receiving pharmacotherapy for Crohn disease but does not remember the name of the medication. Her temperature is 38.2°C (1O0.8aF), pulse is 90/min, respirations are 23/min. and blood pressure is 130/90 mm Hg. Amphoric breath sounds are heard over the right upper lung field. A chest x-ray is shown. This patient is most likely being treated with a monoclonal antibody targeted against which of the following?
A) CD3
B) CD20
C) Complement protein G5
D ) IgE
E) Tumor necrosis factor-a

A

Correct Answer: E.
Monoclonal antibody therapy targeted against tumor necrosis facfor-a (TNF-a) increases a patient’s risk for active mycobacterial infection. This patient presents with fevers: weight loss: and cough; in association with a cavitary right apical lung lesion on chest xray ; which are consistent with a reactivated Mycobacterium tuberculosis (MTB) infection. Monoclonal antibodies against TNF-a that are specifically used to treat Crohn disease include infliximab, adalimumab, and certolizumab. These medications are known to increase the risk for reactivating latent MTB because of their deleterious effect on granuloma formation and maintenance. All patients who are considered for treatment with these agents must undergo screening for latent MTB infection. If positive, treatment for latent MTB with nine months of isoniazid is indicated

Educational Objective: The use of monoclonal antibody therapy against TNF-a is assoc ated with an increased risk for reactivated latent MTB.. Because of this, all prospective candidates for this therapy should be screened for latent MTB and accordingly treated if positive.

56
Q

A 2-year-old boy is brought to the physician because of chronic bacterial respiratory infections since birth. He is currently asymptomatic. Physical examination shows no abnormalities. T- and B-lymphocyte counts and serum antibody concentrations are within the reference ranges. Natural killer cell count and function are normal. Analysis of cellular expression of human leukocyte antigen by flow cytometry shows absence of class I MHC-expressing cells. A diagnosis of bare lymphocyte syndrome, type l; is made. This patient most likely has mutations in the genes encoding which of the following?
A ) Adenosine deaminase
6 } Fas ligand (CD178)
C) lnterleukin-2 (IL-2) receptor a chain (CD25)
D ) Peptide transporter (TAP)

A

Correct Answer: D.
Peptide transporter (TAP) gene mutation accounts for this patient’s diagnosis of bare lymphocyte syndrome, type I There are two TAP genes (1 and 2) that bind iin the endoplasmic reticulum (ER) to form the TAP complex. This protein normally shuttles antigenic
peptides obtained from foreign pathogens into the ER where the antigens are complexed with MHC-I before being shuttled back to the surface of the cell for antigen presentation. This antigen-MHC-l complex is recognized by CD8+ T lymphocytes, which upon recognition of foreign peptides, kill the presenting cell. Absence of TAP results in the failure to present foreign peptides to CD8+ T lymphocytes via MHC- I and thereby results in susceptibility to a wide range of infections. Pulmonary bacterial and skin infections are
most common.

Educational Objective: TAP mutations prevent the presentation of foreign antigens on MHC-I to CDS+ T lymphocytes. This limits the normal immune response resulting in susceptibility to multiple bacterial infections, most commonly pulmonary and skin infections. Patients demonstrate normal numbers of lymphocytes.